Question 4  In this problem we will seek to understand another limitation of trying to use deductive logic to do inductive inference. This is the tacking problem or the problem of irrelevant conjunctions. We say that an inference in an argument is monotonic if adding additional premises to the argument does not make the new argument invalid. For example, we know the inference from F to (F ∨ G) is valid. But if we add any formula H to the premises, then the inference from F and H to (F ∨ G) is still valid because ((F ∧ H) → (F ∨ G)) is a tautology. Using truth tables, show that the following truth-preserving rules of inference are monotonic. In fact, deductive inference is characterized by being monotonic. (a) From F and (F → G), infer G  (b) From (F ∧ G), infer G.  Now, according to the Hypothetico-Deductive (H-D) account of confirmation, evidence E confirms H relative to background knowledge K if and only if: 1. (H ∧ K) is satisfiable. 2. (H ∧ K) entails E. 3. K alone does not entail E. (c) Suppose E confirms H. Here we show that the H-D account of confirmation suffers from the problem of irrelevant conjunctions or the tacking problem. Let H′ = Pa, H = ∀x(Rx → Bx), K = Ra and E = Ba           (i) Show that ((H ∧ H′) ∧ K) is satisfiable by finding a model that makes H, H′ and K true. H′ is the irrelevant conjunction or hypothesis that we have "tacked" to H. It can say anything but here it says that "a is a piece of paper".            (ii) Explain why K alone does not entail E. Hint: Is (K → E) a tautology?            (iii) Since (H ∧ K) entails E, explain why ((H ∧ H′) ∧ K) entails E. Hint: is the inference monotonic? Suppose someone accepts something like the following principle, which we shall call the special conse- quence condition: If E confirms H and H entails H′, then E confirms H′.            (iv) Show that (H ∧ H′) entails H.             (v) Explain why E confirms H′. Hint: Use the special consequence condition. Do you find it plausible that E confirms H′? Why or why not?

Advanced Engineering Mathematics
10th Edition
ISBN:9780470458365
Author:Erwin Kreyszig
Publisher:Erwin Kreyszig
Chapter2: Second-order Linear Odes
Section: Chapter Questions
Problem 1RQ
icon
Related questions
Question
100%

Question 4 

In this problem we will seek to understand another limitation of trying to use deductive logic to do inductive inference. This is the tacking problem or the problem of irrelevant conjunctions. We say that an inference in an argument is monotonic if adding additional premises to the argument does not make the new argument invalid. For example, we know the inference from F to (F ∨ G) is valid. But if we add any formula H to the premises, then the inference from F and H to (F ∨ G) is still valid because

((F ∧ H) → (F ∨ G)) is a tautology.

Using truth tables, show that the following truth-preserving rules of inference are monotonic. In fact, deductive inference is characterized by being monotonic.

(a) From F and (F → G), infer G 

(b) From (F ∧ G), infer G. 

Now, according to the Hypothetico-Deductive (H-D) account of confirmation, evidence E confirms H relative to background knowledge K if and only if:

1. (H ∧ K) is satisfiable.
2. (H ∧ K) entails E.
3. K alone does not entail E.

(c) Suppose E confirms H. Here we show that the H-D account of confirmation suffers from the problem of irrelevant conjunctions or the tacking problem. Let H′ = Pa, H = ∀x(Rx → Bx), K = Ra and E = Ba

          (i) Show that ((H ∧ H′) ∧ K) is satisfiable by finding a model that makes H, H′ and K true. H′ is the irrelevant conjunction or hypothesis that we have "tacked" to H. It can say anything but here it says that "a is a piece of paper". 

          (ii) Explain why K alone does not entail E. Hint: Is (K → E) a tautology? 

          (iii) Since (H ∧ K) entails E, explain why ((H ∧ H′) ∧ K) entails E. Hint: is the inference monotonic?

Suppose someone accepts something like the following principle, which we shall call the special conse- quence condition: If E confirms H and H entails H′, then E confirms H′.

           (iv) Show that (H ∧ H′) entails H. 

           (v) Explain why E confirms H′. Hint: Use the special consequence condition. Do you find it plausible that E confirms H′? Why or why not? 

Expert Solution
trending now

Trending now

This is a popular solution!

steps

Step by step

Solved in 9 steps

Blurred answer
Follow-up Questions
Read through expert solutions to related follow-up questions below.
Follow-up Question

I apologize, for part 4 (c) I typed (iv) incorrectly, there is a "prime" missing. Please correct the answer to the respected problem :

(iv) Show that (H ∧ H′) entails H′ 



 

Solution
Bartleby Expert
SEE SOLUTION
Recommended textbooks for you
Advanced Engineering Mathematics
Advanced Engineering Mathematics
Advanced Math
ISBN:
9780470458365
Author:
Erwin Kreyszig
Publisher:
Wiley, John & Sons, Incorporated
Numerical Methods for Engineers
Numerical Methods for Engineers
Advanced Math
ISBN:
9780073397924
Author:
Steven C. Chapra Dr., Raymond P. Canale
Publisher:
McGraw-Hill Education
Introductory Mathematics for Engineering Applicat…
Introductory Mathematics for Engineering Applicat…
Advanced Math
ISBN:
9781118141809
Author:
Nathan Klingbeil
Publisher:
WILEY
Mathematics For Machine Technology
Mathematics For Machine Technology
Advanced Math
ISBN:
9781337798310
Author:
Peterson, John.
Publisher:
Cengage Learning,
Basic Technical Mathematics
Basic Technical Mathematics
Advanced Math
ISBN:
9780134437705
Author:
Washington
Publisher:
PEARSON
Topology
Topology
Advanced Math
ISBN:
9780134689517
Author:
Munkres, James R.
Publisher:
Pearson,